Difference between revisions of "2002 AIME II Problems/Problem 10"

(Added problem, solution still needed)
Line 3: Line 3:
  
 
== Solution ==
 
== Solution ==
{{solution}}
+
Note that <math>x</math> degrees is equal to <math>\frac{\pi x}{180}</math> radians. Also, for <math>\alpha \in \left[0 , \frac{\pi}{2} \right]</math>, the two least positive angles <math>\theta > \alpha</math> such that <math>\sin{\theta} = \sin{\alpha}</math> are <math>\theta = \pi-\alpha</math>, and <math>\theta = 2\pi + \alpha</math>.
 +
 
 +
Clearly <math>x > \frac{\pi x}{180}</math> for positive real values of <math>x</math>.
 +
 
 +
<math>\theta = \pi-\alpha</math> yields: <math>x = \pi - \frac{\pi x}{180} \Rightarrow x = \frac{180\pi}{180+\pi} \Rightarrow (p,q) = (180,180)</math>.
 +
 
 +
<math>\theta = 2\pi + \alpha</math> yields: <math>x = 2\pi + \frac{\pi x}{180} \Rightarrow x = \frac{360\pi}{180-\pi} \Rightarrow (m,n) = (360,180)</math>.
 +
 
 +
So, <math>m+n+p+q = \boxed{900}</math>.
  
 
== See also ==
 
== See also ==
 
{{AIME box|year=2002|n=II|num-b=9|num-a=11}}
 
{{AIME box|year=2002|n=II|num-b=9|num-a=11}}

Revision as of 23:00, 23 June 2008

Problem

While finding the sine of a certain angle, an absent-minded professor failed to notice that his calculator was not in the correct angular mode. He was lucky to get the right answer. The two least positive real values of $x$ for which the sine of $x$ degrees is the same as the sine of $x$ radians are $\frac{m\pi}{n-\pi}$ and $\frac{p\pi}{q+\pi}$, where $m$, $n$, $p$, and $q$ are positive integers. Find $m+n+p+q$.

Solution

Note that $x$ degrees is equal to $\frac{\pi x}{180}$ radians. Also, for $\alpha \in \left[0 , \frac{\pi}{2} \right]$, the two least positive angles $\theta > \alpha$ such that $\sin{\theta} = \sin{\alpha}$ are $\theta = \pi-\alpha$, and $\theta = 2\pi + \alpha$.

Clearly $x > \frac{\pi x}{180}$ for positive real values of $x$.

$\theta = \pi-\alpha$ yields: $x = \pi - \frac{\pi x}{180} \Rightarrow x = \frac{180\pi}{180+\pi} \Rightarrow (p,q) = (180,180)$.

$\theta = 2\pi + \alpha$ yields: $x = 2\pi + \frac{\pi x}{180} \Rightarrow x = \frac{360\pi}{180-\pi} \Rightarrow (m,n) = (360,180)$.

So, $m+n+p+q = \boxed{900}$.

See also

2002 AIME II (ProblemsAnswer KeyResources)
Preceded by
Problem 9
Followed by
Problem 11
1 2 3 4 5 6 7 8 9 10 11 12 13 14 15
All AIME Problems and Solutions